Is there a set of positive integers of density 1 which contains no infinite arithmetic progression?Density of a set of natural numbers whose differences are not bounded.What is the shortest route to Roth's theorem?Inverse Length 3 Arithmetic Progression Problem for sets with positive upper densityDo there exist sets of integers with arbitrarily large upper density which contains infinitely many elements that are not in an arithmetic progression of length 3?Arithmetic progressions in power sequencesAre there infinite primes among powerful order terms of Dirichlet arithmetic progressions?every arithmetic progression contains a sequence of $k$ “consecutive” primes for possibly all natural numbers $k$?On the upper Banach density of the set of positive integers whose base-$b$ representation misses at least one prescribed digitAdditivity of upper densities with respect to arithmetic progressions of integersExistence of Arithmetic Progression from density inequalityDiscrepancy in non-homogeneous arithmetic progressions

Is there a set of positive integers of density 1 which contains no infinite arithmetic progression?


Density of a set of natural numbers whose differences are not bounded.What is the shortest route to Roth's theorem?Inverse Length 3 Arithmetic Progression Problem for sets with positive upper densityDo there exist sets of integers with arbitrarily large upper density which contains infinitely many elements that are not in an arithmetic progression of length 3?Arithmetic progressions in power sequencesAre there infinite primes among powerful order terms of Dirichlet arithmetic progressions?every arithmetic progression contains a sequence of $k$ “consecutive” primes for possibly all natural numbers $k$?On the upper Banach density of the set of positive integers whose base-$b$ representation misses at least one prescribed digitAdditivity of upper densities with respect to arithmetic progressions of integersExistence of Arithmetic Progression from density inequalityDiscrepancy in non-homogeneous arithmetic progressions













8












$begingroup$


Let $V$ be a set of positive integers whose natural density is 1. Is it necessarily true that $V$ contains an infinite arithmetic progression?—i.e., that there are non-negative integers $a,b,nu$ with $0leq bleq a-1$ so that: $$left an+b:ngeqnuright subseteq V$$



In addition to an answer, any references on the matter would be most appreciated.










share|cite|improve this question









$endgroup$







  • 26




    $begingroup$
    Enumerate all the arithmetic progressions as $A_n$. Now remove from $mathbb N$ the $2^n$-th element of $A_n$ for each $n$.
    $endgroup$
    – Wojowu
    Jun 6 at 21:33






  • 1




    $begingroup$
    So sets that contradict this exist? Excellent! Thank you. :)
    $endgroup$
    – MCS
    Jun 6 at 21:35







  • 8




    $begingroup$
    Generalizing Wojowu’s construction shows that “density 1” can be achieved in the strongest possible way: for any function $f$ that tends to infinity, no matter how slowly, there is a set of non-negative integers containing at least $n-f(n)$ integers between $1$ and $n$ for each $n$ but no infinite arithmetic progression.
    $endgroup$
    – Henry Cohn
    Jun 6 at 22:49






  • 7




    $begingroup$
    A more constructive way would be the following: Take any increasing function $f$, and put $V=mathbbNsetminusbigcup_ngeq 1 [f(n), f(n)+n]$. If $f$ grows faster than $n^2$, than $V$ has density 1. By increasing the growth of $f$ you can make $mathbbNsetminus V$ arbitrary small.
    $endgroup$
    – Jan-Christoph Schlage-Puchta
    Jun 7 at 12:35











  • $begingroup$
    The answer I had in mind is already written up here. This example arises in a different context [finding arbitrarily long strings of consecutive composite numbers] and/but satisfies the criterion here, too.
    $endgroup$
    – Benjamin Dickman
    Jun 9 at 18:45















8












$begingroup$


Let $V$ be a set of positive integers whose natural density is 1. Is it necessarily true that $V$ contains an infinite arithmetic progression?—i.e., that there are non-negative integers $a,b,nu$ with $0leq bleq a-1$ so that: $$left an+b:ngeqnuright subseteq V$$



In addition to an answer, any references on the matter would be most appreciated.










share|cite|improve this question









$endgroup$







  • 26




    $begingroup$
    Enumerate all the arithmetic progressions as $A_n$. Now remove from $mathbb N$ the $2^n$-th element of $A_n$ for each $n$.
    $endgroup$
    – Wojowu
    Jun 6 at 21:33






  • 1




    $begingroup$
    So sets that contradict this exist? Excellent! Thank you. :)
    $endgroup$
    – MCS
    Jun 6 at 21:35







  • 8




    $begingroup$
    Generalizing Wojowu’s construction shows that “density 1” can be achieved in the strongest possible way: for any function $f$ that tends to infinity, no matter how slowly, there is a set of non-negative integers containing at least $n-f(n)$ integers between $1$ and $n$ for each $n$ but no infinite arithmetic progression.
    $endgroup$
    – Henry Cohn
    Jun 6 at 22:49






  • 7




    $begingroup$
    A more constructive way would be the following: Take any increasing function $f$, and put $V=mathbbNsetminusbigcup_ngeq 1 [f(n), f(n)+n]$. If $f$ grows faster than $n^2$, than $V$ has density 1. By increasing the growth of $f$ you can make $mathbbNsetminus V$ arbitrary small.
    $endgroup$
    – Jan-Christoph Schlage-Puchta
    Jun 7 at 12:35











  • $begingroup$
    The answer I had in mind is already written up here. This example arises in a different context [finding arbitrarily long strings of consecutive composite numbers] and/but satisfies the criterion here, too.
    $endgroup$
    – Benjamin Dickman
    Jun 9 at 18:45













8












8








8


4



$begingroup$


Let $V$ be a set of positive integers whose natural density is 1. Is it necessarily true that $V$ contains an infinite arithmetic progression?—i.e., that there are non-negative integers $a,b,nu$ with $0leq bleq a-1$ so that: $$left an+b:ngeqnuright subseteq V$$



In addition to an answer, any references on the matter would be most appreciated.










share|cite|improve this question









$endgroup$




Let $V$ be a set of positive integers whose natural density is 1. Is it necessarily true that $V$ contains an infinite arithmetic progression?—i.e., that there are non-negative integers $a,b,nu$ with $0leq bleq a-1$ so that: $$left an+b:ngeqnuright subseteq V$$



In addition to an answer, any references on the matter would be most appreciated.







nt.number-theory additive-combinatorics






share|cite|improve this question













share|cite|improve this question











share|cite|improve this question




share|cite|improve this question










asked Jun 6 at 21:28









MCSMCS

313111




313111







  • 26




    $begingroup$
    Enumerate all the arithmetic progressions as $A_n$. Now remove from $mathbb N$ the $2^n$-th element of $A_n$ for each $n$.
    $endgroup$
    – Wojowu
    Jun 6 at 21:33






  • 1




    $begingroup$
    So sets that contradict this exist? Excellent! Thank you. :)
    $endgroup$
    – MCS
    Jun 6 at 21:35







  • 8




    $begingroup$
    Generalizing Wojowu’s construction shows that “density 1” can be achieved in the strongest possible way: for any function $f$ that tends to infinity, no matter how slowly, there is a set of non-negative integers containing at least $n-f(n)$ integers between $1$ and $n$ for each $n$ but no infinite arithmetic progression.
    $endgroup$
    – Henry Cohn
    Jun 6 at 22:49






  • 7




    $begingroup$
    A more constructive way would be the following: Take any increasing function $f$, and put $V=mathbbNsetminusbigcup_ngeq 1 [f(n), f(n)+n]$. If $f$ grows faster than $n^2$, than $V$ has density 1. By increasing the growth of $f$ you can make $mathbbNsetminus V$ arbitrary small.
    $endgroup$
    – Jan-Christoph Schlage-Puchta
    Jun 7 at 12:35











  • $begingroup$
    The answer I had in mind is already written up here. This example arises in a different context [finding arbitrarily long strings of consecutive composite numbers] and/but satisfies the criterion here, too.
    $endgroup$
    – Benjamin Dickman
    Jun 9 at 18:45












  • 26




    $begingroup$
    Enumerate all the arithmetic progressions as $A_n$. Now remove from $mathbb N$ the $2^n$-th element of $A_n$ for each $n$.
    $endgroup$
    – Wojowu
    Jun 6 at 21:33






  • 1




    $begingroup$
    So sets that contradict this exist? Excellent! Thank you. :)
    $endgroup$
    – MCS
    Jun 6 at 21:35







  • 8




    $begingroup$
    Generalizing Wojowu’s construction shows that “density 1” can be achieved in the strongest possible way: for any function $f$ that tends to infinity, no matter how slowly, there is a set of non-negative integers containing at least $n-f(n)$ integers between $1$ and $n$ for each $n$ but no infinite arithmetic progression.
    $endgroup$
    – Henry Cohn
    Jun 6 at 22:49






  • 7




    $begingroup$
    A more constructive way would be the following: Take any increasing function $f$, and put $V=mathbbNsetminusbigcup_ngeq 1 [f(n), f(n)+n]$. If $f$ grows faster than $n^2$, than $V$ has density 1. By increasing the growth of $f$ you can make $mathbbNsetminus V$ arbitrary small.
    $endgroup$
    – Jan-Christoph Schlage-Puchta
    Jun 7 at 12:35











  • $begingroup$
    The answer I had in mind is already written up here. This example arises in a different context [finding arbitrarily long strings of consecutive composite numbers] and/but satisfies the criterion here, too.
    $endgroup$
    – Benjamin Dickman
    Jun 9 at 18:45







26




26




$begingroup$
Enumerate all the arithmetic progressions as $A_n$. Now remove from $mathbb N$ the $2^n$-th element of $A_n$ for each $n$.
$endgroup$
– Wojowu
Jun 6 at 21:33




$begingroup$
Enumerate all the arithmetic progressions as $A_n$. Now remove from $mathbb N$ the $2^n$-th element of $A_n$ for each $n$.
$endgroup$
– Wojowu
Jun 6 at 21:33




1




1




$begingroup$
So sets that contradict this exist? Excellent! Thank you. :)
$endgroup$
– MCS
Jun 6 at 21:35





$begingroup$
So sets that contradict this exist? Excellent! Thank you. :)
$endgroup$
– MCS
Jun 6 at 21:35





8




8




$begingroup$
Generalizing Wojowu’s construction shows that “density 1” can be achieved in the strongest possible way: for any function $f$ that tends to infinity, no matter how slowly, there is a set of non-negative integers containing at least $n-f(n)$ integers between $1$ and $n$ for each $n$ but no infinite arithmetic progression.
$endgroup$
– Henry Cohn
Jun 6 at 22:49




$begingroup$
Generalizing Wojowu’s construction shows that “density 1” can be achieved in the strongest possible way: for any function $f$ that tends to infinity, no matter how slowly, there is a set of non-negative integers containing at least $n-f(n)$ integers between $1$ and $n$ for each $n$ but no infinite arithmetic progression.
$endgroup$
– Henry Cohn
Jun 6 at 22:49




7




7




$begingroup$
A more constructive way would be the following: Take any increasing function $f$, and put $V=mathbbNsetminusbigcup_ngeq 1 [f(n), f(n)+n]$. If $f$ grows faster than $n^2$, than $V$ has density 1. By increasing the growth of $f$ you can make $mathbbNsetminus V$ arbitrary small.
$endgroup$
– Jan-Christoph Schlage-Puchta
Jun 7 at 12:35





$begingroup$
A more constructive way would be the following: Take any increasing function $f$, and put $V=mathbbNsetminusbigcup_ngeq 1 [f(n), f(n)+n]$. If $f$ grows faster than $n^2$, than $V$ has density 1. By increasing the growth of $f$ you can make $mathbbNsetminus V$ arbitrary small.
$endgroup$
– Jan-Christoph Schlage-Puchta
Jun 7 at 12:35













$begingroup$
The answer I had in mind is already written up here. This example arises in a different context [finding arbitrarily long strings of consecutive composite numbers] and/but satisfies the criterion here, too.
$endgroup$
– Benjamin Dickman
Jun 9 at 18:45




$begingroup$
The answer I had in mind is already written up here. This example arises in a different context [finding arbitrarily long strings of consecutive composite numbers] and/but satisfies the criterion here, too.
$endgroup$
– Benjamin Dickman
Jun 9 at 18:45










3 Answers
3






active

oldest

votes


















11












$begingroup$

Another construction is to let $n notin V$ if and only if $n$ begins with at least $sqrtlogn$ consecutive '9's when written in decimal. This satisfies the stronger property that there is no non-constant polynomial $f : mathbbN rightarrow mathbbN$ whose image is contained entirely in $V$.






share|cite|improve this answer









$endgroup$




















    21












    $begingroup$

    Here is a concrete counterexample (where $mathbbN$ is the set of positive integers):
    $$V:=mathbbNsetminusa^2b^2+b:a,binmathbbN.$$
    It is straightforward to see that $V$ has density $1$, but it does not contain an infinite arithmetic progression.






    share|cite|improve this answer









    $endgroup$








    • 6




      $begingroup$
      Here is a straightforward proof for anyone else who found the density non-obvious: In $(a,b):a^2b^2+b<N$, each value of $b$ has at most $sqrtN/b$ values of $a$. Also $b<sqrtN$. So the cardinality of the set is at most $$sum_b=1^sqrtN fracsqrtNb < 2int_1^sqrtN+1fracsqrtNbdb< 2sqrtNln(N)$$ Since $2sqrtNln(N)/Nrightarrow 0$, the density of these sets goes to 0.
      $endgroup$
      – Matt F.
      Jun 7 at 19:04


















    17












    $begingroup$

    Yet another concrete counterexample:
    $$ bigcup_n=1^infty [n^3+n,(n+1)^3]. $$
    More generally, any set containing arbitrarily long gaps is free of infinite arithmetic progressions, and has natural density $1$ if the gaps are properly spaced.



    Incidentally, an incomparably subtler question is whether any set of positive natural density contains arbitrarily long arithmetic progressions. This was conjecture by Erdős and Turán in 1936 and famously proved by Szemerédi almost 40 years later.






    share|cite|improve this answer











    $endgroup$








    • 5




      $begingroup$
      Szemeredi theorem asks for sets with positive natural density. For density $1$ the problem is almost trivial - if a set doesn't contain a $k$-AP for some $k$, it in particular contains no block of $k$ consecutive elements, which implies it has density at most $k/(k+1)$.
      $endgroup$
      – Wojowu
      Jun 7 at 12:45






    • 1




      $begingroup$
      I will add a link to an older post related to arbitrarily long gaps and densities: Density of a set of natural numbers whose differences are not bounded.
      $endgroup$
      – Martin Sleziak
      Jun 7 at 12:51






    • 1




      $begingroup$
      @Wojowu: absolutely - my bad; I have edited the answer.
      $endgroup$
      – Seva
      Jun 7 at 12:58






    • 2




      $begingroup$
      @Wojowu Doesn't Szemeredi's theorem just require positve upper density?
      $endgroup$
      – bof
      Jun 7 at 15:46











    Your Answer








    StackExchange.ready(function()
    var channelOptions =
    tags: "".split(" "),
    id: "504"
    ;
    initTagRenderer("".split(" "), "".split(" "), channelOptions);

    StackExchange.using("externalEditor", function()
    // Have to fire editor after snippets, if snippets enabled
    if (StackExchange.settings.snippets.snippetsEnabled)
    StackExchange.using("snippets", function()
    createEditor();
    );

    else
    createEditor();

    );

    function createEditor()
    StackExchange.prepareEditor(
    heartbeatType: 'answer',
    autoActivateHeartbeat: false,
    convertImagesToLinks: true,
    noModals: true,
    showLowRepImageUploadWarning: true,
    reputationToPostImages: 10,
    bindNavPrevention: true,
    postfix: "",
    imageUploader:
    brandingHtml: "Powered by u003ca class="icon-imgur-white" href="https://imgur.com/"u003eu003c/au003e",
    contentPolicyHtml: "User contributions licensed under u003ca href="https://creativecommons.org/licenses/by-sa/3.0/"u003ecc by-sa 3.0 with attribution requiredu003c/au003e u003ca href="https://stackoverflow.com/legal/content-policy"u003e(content policy)u003c/au003e",
    allowUrls: true
    ,
    noCode: true, onDemand: true,
    discardSelector: ".discard-answer"
    ,immediatelyShowMarkdownHelp:true
    );



    );













    draft saved

    draft discarded


















    StackExchange.ready(
    function ()
    StackExchange.openid.initPostLogin('.new-post-login', 'https%3a%2f%2fmathoverflow.net%2fquestions%2f333430%2fis-there-a-set-of-positive-integers-of-density-1-which-contains-no-infinite-arit%23new-answer', 'question_page');

    );

    Post as a guest















    Required, but never shown

























    3 Answers
    3






    active

    oldest

    votes








    3 Answers
    3






    active

    oldest

    votes









    active

    oldest

    votes






    active

    oldest

    votes









    11












    $begingroup$

    Another construction is to let $n notin V$ if and only if $n$ begins with at least $sqrtlogn$ consecutive '9's when written in decimal. This satisfies the stronger property that there is no non-constant polynomial $f : mathbbN rightarrow mathbbN$ whose image is contained entirely in $V$.






    share|cite|improve this answer









    $endgroup$

















      11












      $begingroup$

      Another construction is to let $n notin V$ if and only if $n$ begins with at least $sqrtlogn$ consecutive '9's when written in decimal. This satisfies the stronger property that there is no non-constant polynomial $f : mathbbN rightarrow mathbbN$ whose image is contained entirely in $V$.






      share|cite|improve this answer









      $endgroup$















        11












        11








        11





        $begingroup$

        Another construction is to let $n notin V$ if and only if $n$ begins with at least $sqrtlogn$ consecutive '9's when written in decimal. This satisfies the stronger property that there is no non-constant polynomial $f : mathbbN rightarrow mathbbN$ whose image is contained entirely in $V$.






        share|cite|improve this answer









        $endgroup$



        Another construction is to let $n notin V$ if and only if $n$ begins with at least $sqrtlogn$ consecutive '9's when written in decimal. This satisfies the stronger property that there is no non-constant polynomial $f : mathbbN rightarrow mathbbN$ whose image is contained entirely in $V$.







        share|cite|improve this answer












        share|cite|improve this answer



        share|cite|improve this answer










        answered Jun 7 at 13:34









        Adam P. GoucherAdam P. Goucher

        7,23523263




        7,23523263





















            21












            $begingroup$

            Here is a concrete counterexample (where $mathbbN$ is the set of positive integers):
            $$V:=mathbbNsetminusa^2b^2+b:a,binmathbbN.$$
            It is straightforward to see that $V$ has density $1$, but it does not contain an infinite arithmetic progression.






            share|cite|improve this answer









            $endgroup$








            • 6




              $begingroup$
              Here is a straightforward proof for anyone else who found the density non-obvious: In $(a,b):a^2b^2+b<N$, each value of $b$ has at most $sqrtN/b$ values of $a$. Also $b<sqrtN$. So the cardinality of the set is at most $$sum_b=1^sqrtN fracsqrtNb < 2int_1^sqrtN+1fracsqrtNbdb< 2sqrtNln(N)$$ Since $2sqrtNln(N)/Nrightarrow 0$, the density of these sets goes to 0.
              $endgroup$
              – Matt F.
              Jun 7 at 19:04















            21












            $begingroup$

            Here is a concrete counterexample (where $mathbbN$ is the set of positive integers):
            $$V:=mathbbNsetminusa^2b^2+b:a,binmathbbN.$$
            It is straightforward to see that $V$ has density $1$, but it does not contain an infinite arithmetic progression.






            share|cite|improve this answer









            $endgroup$








            • 6




              $begingroup$
              Here is a straightforward proof for anyone else who found the density non-obvious: In $(a,b):a^2b^2+b<N$, each value of $b$ has at most $sqrtN/b$ values of $a$. Also $b<sqrtN$. So the cardinality of the set is at most $$sum_b=1^sqrtN fracsqrtNb < 2int_1^sqrtN+1fracsqrtNbdb< 2sqrtNln(N)$$ Since $2sqrtNln(N)/Nrightarrow 0$, the density of these sets goes to 0.
              $endgroup$
              – Matt F.
              Jun 7 at 19:04













            21












            21








            21





            $begingroup$

            Here is a concrete counterexample (where $mathbbN$ is the set of positive integers):
            $$V:=mathbbNsetminusa^2b^2+b:a,binmathbbN.$$
            It is straightforward to see that $V$ has density $1$, but it does not contain an infinite arithmetic progression.






            share|cite|improve this answer









            $endgroup$



            Here is a concrete counterexample (where $mathbbN$ is the set of positive integers):
            $$V:=mathbbNsetminusa^2b^2+b:a,binmathbbN.$$
            It is straightforward to see that $V$ has density $1$, but it does not contain an infinite arithmetic progression.







            share|cite|improve this answer












            share|cite|improve this answer



            share|cite|improve this answer










            answered Jun 6 at 22:01









            GH from MOGH from MO

            61.1k5154234




            61.1k5154234







            • 6




              $begingroup$
              Here is a straightforward proof for anyone else who found the density non-obvious: In $(a,b):a^2b^2+b<N$, each value of $b$ has at most $sqrtN/b$ values of $a$. Also $b<sqrtN$. So the cardinality of the set is at most $$sum_b=1^sqrtN fracsqrtNb < 2int_1^sqrtN+1fracsqrtNbdb< 2sqrtNln(N)$$ Since $2sqrtNln(N)/Nrightarrow 0$, the density of these sets goes to 0.
              $endgroup$
              – Matt F.
              Jun 7 at 19:04












            • 6




              $begingroup$
              Here is a straightforward proof for anyone else who found the density non-obvious: In $(a,b):a^2b^2+b<N$, each value of $b$ has at most $sqrtN/b$ values of $a$. Also $b<sqrtN$. So the cardinality of the set is at most $$sum_b=1^sqrtN fracsqrtNb < 2int_1^sqrtN+1fracsqrtNbdb< 2sqrtNln(N)$$ Since $2sqrtNln(N)/Nrightarrow 0$, the density of these sets goes to 0.
              $endgroup$
              – Matt F.
              Jun 7 at 19:04







            6




            6




            $begingroup$
            Here is a straightforward proof for anyone else who found the density non-obvious: In $(a,b):a^2b^2+b<N$, each value of $b$ has at most $sqrtN/b$ values of $a$. Also $b<sqrtN$. So the cardinality of the set is at most $$sum_b=1^sqrtN fracsqrtNb < 2int_1^sqrtN+1fracsqrtNbdb< 2sqrtNln(N)$$ Since $2sqrtNln(N)/Nrightarrow 0$, the density of these sets goes to 0.
            $endgroup$
            – Matt F.
            Jun 7 at 19:04




            $begingroup$
            Here is a straightforward proof for anyone else who found the density non-obvious: In $(a,b):a^2b^2+b<N$, each value of $b$ has at most $sqrtN/b$ values of $a$. Also $b<sqrtN$. So the cardinality of the set is at most $$sum_b=1^sqrtN fracsqrtNb < 2int_1^sqrtN+1fracsqrtNbdb< 2sqrtNln(N)$$ Since $2sqrtNln(N)/Nrightarrow 0$, the density of these sets goes to 0.
            $endgroup$
            – Matt F.
            Jun 7 at 19:04











            17












            $begingroup$

            Yet another concrete counterexample:
            $$ bigcup_n=1^infty [n^3+n,(n+1)^3]. $$
            More generally, any set containing arbitrarily long gaps is free of infinite arithmetic progressions, and has natural density $1$ if the gaps are properly spaced.



            Incidentally, an incomparably subtler question is whether any set of positive natural density contains arbitrarily long arithmetic progressions. This was conjecture by Erdős and Turán in 1936 and famously proved by Szemerédi almost 40 years later.






            share|cite|improve this answer











            $endgroup$








            • 5




              $begingroup$
              Szemeredi theorem asks for sets with positive natural density. For density $1$ the problem is almost trivial - if a set doesn't contain a $k$-AP for some $k$, it in particular contains no block of $k$ consecutive elements, which implies it has density at most $k/(k+1)$.
              $endgroup$
              – Wojowu
              Jun 7 at 12:45






            • 1




              $begingroup$
              I will add a link to an older post related to arbitrarily long gaps and densities: Density of a set of natural numbers whose differences are not bounded.
              $endgroup$
              – Martin Sleziak
              Jun 7 at 12:51






            • 1




              $begingroup$
              @Wojowu: absolutely - my bad; I have edited the answer.
              $endgroup$
              – Seva
              Jun 7 at 12:58






            • 2




              $begingroup$
              @Wojowu Doesn't Szemeredi's theorem just require positve upper density?
              $endgroup$
              – bof
              Jun 7 at 15:46















            17












            $begingroup$

            Yet another concrete counterexample:
            $$ bigcup_n=1^infty [n^3+n,(n+1)^3]. $$
            More generally, any set containing arbitrarily long gaps is free of infinite arithmetic progressions, and has natural density $1$ if the gaps are properly spaced.



            Incidentally, an incomparably subtler question is whether any set of positive natural density contains arbitrarily long arithmetic progressions. This was conjecture by Erdős and Turán in 1936 and famously proved by Szemerédi almost 40 years later.






            share|cite|improve this answer











            $endgroup$








            • 5




              $begingroup$
              Szemeredi theorem asks for sets with positive natural density. For density $1$ the problem is almost trivial - if a set doesn't contain a $k$-AP for some $k$, it in particular contains no block of $k$ consecutive elements, which implies it has density at most $k/(k+1)$.
              $endgroup$
              – Wojowu
              Jun 7 at 12:45






            • 1




              $begingroup$
              I will add a link to an older post related to arbitrarily long gaps and densities: Density of a set of natural numbers whose differences are not bounded.
              $endgroup$
              – Martin Sleziak
              Jun 7 at 12:51






            • 1




              $begingroup$
              @Wojowu: absolutely - my bad; I have edited the answer.
              $endgroup$
              – Seva
              Jun 7 at 12:58






            • 2




              $begingroup$
              @Wojowu Doesn't Szemeredi's theorem just require positve upper density?
              $endgroup$
              – bof
              Jun 7 at 15:46













            17












            17








            17





            $begingroup$

            Yet another concrete counterexample:
            $$ bigcup_n=1^infty [n^3+n,(n+1)^3]. $$
            More generally, any set containing arbitrarily long gaps is free of infinite arithmetic progressions, and has natural density $1$ if the gaps are properly spaced.



            Incidentally, an incomparably subtler question is whether any set of positive natural density contains arbitrarily long arithmetic progressions. This was conjecture by Erdős and Turán in 1936 and famously proved by Szemerédi almost 40 years later.






            share|cite|improve this answer











            $endgroup$



            Yet another concrete counterexample:
            $$ bigcup_n=1^infty [n^3+n,(n+1)^3]. $$
            More generally, any set containing arbitrarily long gaps is free of infinite arithmetic progressions, and has natural density $1$ if the gaps are properly spaced.



            Incidentally, an incomparably subtler question is whether any set of positive natural density contains arbitrarily long arithmetic progressions. This was conjecture by Erdős and Turán in 1936 and famously proved by Szemerédi almost 40 years later.







            share|cite|improve this answer














            share|cite|improve this answer



            share|cite|improve this answer








            edited Jun 7 at 20:11









            GH from MO

            61.1k5154234




            61.1k5154234










            answered Jun 7 at 12:28









            SevaSeva

            13.5k138105




            13.5k138105







            • 5




              $begingroup$
              Szemeredi theorem asks for sets with positive natural density. For density $1$ the problem is almost trivial - if a set doesn't contain a $k$-AP for some $k$, it in particular contains no block of $k$ consecutive elements, which implies it has density at most $k/(k+1)$.
              $endgroup$
              – Wojowu
              Jun 7 at 12:45






            • 1




              $begingroup$
              I will add a link to an older post related to arbitrarily long gaps and densities: Density of a set of natural numbers whose differences are not bounded.
              $endgroup$
              – Martin Sleziak
              Jun 7 at 12:51






            • 1




              $begingroup$
              @Wojowu: absolutely - my bad; I have edited the answer.
              $endgroup$
              – Seva
              Jun 7 at 12:58






            • 2




              $begingroup$
              @Wojowu Doesn't Szemeredi's theorem just require positve upper density?
              $endgroup$
              – bof
              Jun 7 at 15:46












            • 5




              $begingroup$
              Szemeredi theorem asks for sets with positive natural density. For density $1$ the problem is almost trivial - if a set doesn't contain a $k$-AP for some $k$, it in particular contains no block of $k$ consecutive elements, which implies it has density at most $k/(k+1)$.
              $endgroup$
              – Wojowu
              Jun 7 at 12:45






            • 1




              $begingroup$
              I will add a link to an older post related to arbitrarily long gaps and densities: Density of a set of natural numbers whose differences are not bounded.
              $endgroup$
              – Martin Sleziak
              Jun 7 at 12:51






            • 1




              $begingroup$
              @Wojowu: absolutely - my bad; I have edited the answer.
              $endgroup$
              – Seva
              Jun 7 at 12:58






            • 2




              $begingroup$
              @Wojowu Doesn't Szemeredi's theorem just require positve upper density?
              $endgroup$
              – bof
              Jun 7 at 15:46







            5




            5




            $begingroup$
            Szemeredi theorem asks for sets with positive natural density. For density $1$ the problem is almost trivial - if a set doesn't contain a $k$-AP for some $k$, it in particular contains no block of $k$ consecutive elements, which implies it has density at most $k/(k+1)$.
            $endgroup$
            – Wojowu
            Jun 7 at 12:45




            $begingroup$
            Szemeredi theorem asks for sets with positive natural density. For density $1$ the problem is almost trivial - if a set doesn't contain a $k$-AP for some $k$, it in particular contains no block of $k$ consecutive elements, which implies it has density at most $k/(k+1)$.
            $endgroup$
            – Wojowu
            Jun 7 at 12:45




            1




            1




            $begingroup$
            I will add a link to an older post related to arbitrarily long gaps and densities: Density of a set of natural numbers whose differences are not bounded.
            $endgroup$
            – Martin Sleziak
            Jun 7 at 12:51




            $begingroup$
            I will add a link to an older post related to arbitrarily long gaps and densities: Density of a set of natural numbers whose differences are not bounded.
            $endgroup$
            – Martin Sleziak
            Jun 7 at 12:51




            1




            1




            $begingroup$
            @Wojowu: absolutely - my bad; I have edited the answer.
            $endgroup$
            – Seva
            Jun 7 at 12:58




            $begingroup$
            @Wojowu: absolutely - my bad; I have edited the answer.
            $endgroup$
            – Seva
            Jun 7 at 12:58




            2




            2




            $begingroup$
            @Wojowu Doesn't Szemeredi's theorem just require positve upper density?
            $endgroup$
            – bof
            Jun 7 at 15:46




            $begingroup$
            @Wojowu Doesn't Szemeredi's theorem just require positve upper density?
            $endgroup$
            – bof
            Jun 7 at 15:46

















            draft saved

            draft discarded
















































            Thanks for contributing an answer to MathOverflow!


            • Please be sure to answer the question. Provide details and share your research!

            But avoid


            • Asking for help, clarification, or responding to other answers.

            • Making statements based on opinion; back them up with references or personal experience.

            Use MathJax to format equations. MathJax reference.


            To learn more, see our tips on writing great answers.




            draft saved


            draft discarded














            StackExchange.ready(
            function ()
            StackExchange.openid.initPostLogin('.new-post-login', 'https%3a%2f%2fmathoverflow.net%2fquestions%2f333430%2fis-there-a-set-of-positive-integers-of-density-1-which-contains-no-infinite-arit%23new-answer', 'question_page');

            );

            Post as a guest















            Required, but never shown





















































            Required, but never shown














            Required, but never shown












            Required, but never shown







            Required, but never shown

































            Required, but never shown














            Required, but never shown












            Required, but never shown







            Required, but never shown







            Popular posts from this blog

            Get product attribute by attribute group code in magento 2get product attribute by product attribute group in magento 2Magento 2 Log Bundle Product Data in List Page?How to get all product attribute of a attribute group of Default attribute set?Magento 2.1 Create a filter in the product grid by new attributeMagento 2 : Get Product Attribute values By GroupMagento 2 How to get all existing values for one attributeMagento 2 get custom attribute of a single product inside a pluginMagento 2.3 How to get all the Multi Source Inventory (MSI) locations collection in custom module?Magento2: how to develop rest API to get new productsGet product attribute by attribute group code ( [attribute_group_code] ) in magento 2

            Category:9 (number) SubcategoriesMedia in category "9 (number)"Navigation menuUpload mediaGND ID: 4485639-8Library of Congress authority ID: sh85091979ReasonatorScholiaStatistics

            Magento 2.3: How do i solve this, Not registered handle, on custom form?How can i rewrite TierPrice Block in Magento2magento 2 captcha not rendering if I override layout xmlmain.CRITICAL: Plugin class doesn't existMagento 2 : Problem while adding custom button order view page?Magento 2.2.5: Overriding Admin Controller sales/orderMagento 2.2.5: Add, Update and Delete existing products Custom OptionsMagento 2.3 : File Upload issue in UI Component FormMagento2 Not registered handleHow to configured Form Builder Js in my custom magento 2.3.0 module?Magento 2.3. How to create image upload field in an admin form